www.matheraum.de
Das Matheforum.
Das Matheforum des MatheRaum.

Für Schüler, Studenten, Lehrer, Mathematik-Interessierte.
Hallo Gast!einloggen | registrieren ]
Startseite · Forum · Wissen · Kurse · Mitglieder · Team · Impressum
Forenbaum
^ Forenbaum
Status Mathe
  Status Schulmathe
    Status Primarstufe
    Status Mathe Klassen 5-7
    Status Mathe Klassen 8-10
    Status Oberstufenmathe
    Status Mathe-Wettbewerbe
    Status Sonstiges
  Status Hochschulmathe
    Status Uni-Analysis
    Status Uni-Lin. Algebra
    Status Algebra+Zahlentheo.
    Status Diskrete Mathematik
    Status Fachdidaktik
    Status Finanz+Versicherung
    Status Logik+Mengenlehre
    Status Numerik
    Status Uni-Stochastik
    Status Topologie+Geometrie
    Status Uni-Sonstiges
  Status Mathe-Vorkurse
    Status Organisatorisches
    Status Schule
    Status Universität
  Status Mathe-Software
    Status Derive
    Status DynaGeo
    Status FunkyPlot
    Status GeoGebra
    Status LaTeX
    Status Maple
    Status MathCad
    Status Mathematica
    Status Matlab
    Status Maxima
    Status MuPad
    Status Taschenrechner

Gezeigt werden alle Foren bis zur Tiefe 2

Navigation
 Startseite...
 Neuerdings beta neu
 Forum...
 vorwissen...
 vorkurse...
 Werkzeuge...
 Nachhilfevermittlung beta...
 Online-Spiele beta
 Suchen
 Verein...
 Impressum
Das Projekt
Server und Internetanbindung werden durch Spenden finanziert.
Organisiert wird das Projekt von unserem Koordinatorenteam.
Hunderte Mitglieder helfen ehrenamtlich in unseren moderierten Foren.
Anbieter der Seite ist der gemeinnützige Verein "Vorhilfe.de e.V.".
Partnerseiten
Mathe-Seiten:Weitere Fächer:

Open Source FunktionenplotterFunkyPlot: Kostenloser und quelloffener Funktionenplotter für Linux und andere Betriebssysteme
StartseiteMatheForenReelle Analysis mehrerer VeränderlichenSatz vom Maximum und Mini
Foren für weitere Schulfächer findest Du auf www.vorhilfe.de z.B. Philosophie • Religion • Kunst • Musik • Sport • Pädagogik
Forum "Reelle Analysis mehrerer Veränderlichen" - Satz vom Maximum und Mini
Satz vom Maximum und Mini < mehrere Veränderl. < reell < Analysis < Hochschule < Mathe < Vorhilfe
Ansicht: [ geschachtelt ] | ^ Forum "Reelle Analysis mehrerer Veränderlichen"  | ^^ Alle Foren  | ^ Forenbaum  | Materialien

Satz vom Maximum und Mini: Frage (beantwortet)
Status: (Frage) beantwortet Status 
Datum: 18:21 So 02.06.2013
Autor: Joker08

Aufgabe
Sei (X,d) metrischer Raum.

(a) Zeigen Sie: Für Festes [mm] a\in [/mm] X ist die Abbildung f: [mm] X\to \IR [/mm] mit f(x):=d(x,a) stetig.

(b) Sie K [mm] \subset [/mm] X kompakt. Zeigen Sie, dass es zu jedem [mm] a\in [/mm] X einen Punkt [mm] b\in [/mm] K mit d(a,b)=dist(a,K) gibt, also einen "nächsten Punkt" zu a in K.

(c) Dieser ist sogar eindeutig, wenn [mm] X=\IR^n [/mm] und zusätzlich K konvex ist.


Also Aufgabenteil (a) und (b) habe ich bereits erledigt.

Ich hänge nun bei aufgabenteil (c).


Also eine Teilmenge K eines Vektorraums E über [mm] \IR [/mm] heißt konvex, falls mit je zwei Punkten x,y [mm] \in [/mm] K auch die Verbindungsstrecke

[mm] [x,y]:=\{(1-t)x+ty:0\leq t\leq 1 \} [/mm]

in K liegt.

Auf [mm] \IR^n [/mm] habe ich dann ja dann ja die euklidische Metrik, also f(x):=|x-a|

Ich sehe aber noch nicht, wie mich die konvexität hier weiterbringen soll :|

        
Bezug
Satz vom Maximum und Mini: Antwort
Status: (Antwort) fertig Status 
Datum: 19:37 So 02.06.2013
Autor: Marcel

Hallo,

> Sei (X,d) metrischer Raum.
>  
> (a) Zeigen Sie: Für Festes [mm]a\in[/mm] X ist die Abbildung f:
> [mm]X\to \IR[/mm] mit f(x):=d(x,a) stetig.
>  
> (b) Sie K [mm]\subset[/mm] X kompakt. Zeigen Sie, dass es zu jedem
> [mm]a\in[/mm] X einen Punkt [mm]b\in[/mm] K mit d(a,b)=dist(a,K) gibt, also
> einen "nächsten Punkt" zu a in K.
>  
> (c) Dieser ist sogar eindeutig, wenn [mm]X=\IR^n[/mm] und
> zusätzlich K konvex ist.
>  
> Also Aufgabenteil (a) und (b) habe ich bereits erledigt.
>  
> Ich hänge nun bei aufgabenteil (c).
>  
>
> Also eine Teilmenge K eines Vektorraums E über [mm]\IR[/mm] heißt
> konvex, falls mit je zwei Punkten x,y [mm]\in[/mm] K auch die
> Verbindungsstrecke
>  
> [mm][x,y]:=\{(1-t)x+ty:0\leq t\leq 1 \}[/mm]
>  
> in K liegt.
>  
> Auf [mm]\IR^n[/mm] habe ich dann ja dann ja die euklidische Metrik,
> also f(x):=|x-a|
>  
> Ich sehe aber noch nicht, wie mich die konvexität hier
> weiterbringen soll :|

na, ich kenne die Aufgabe jetzt auch nicht auswendig und ich habe sie mir
auch nicht neu und zu Ende überlegt, aber was liegt denn näher, als hier
so anzufangen:
Angenommen, es gäbe zwei Punkte $b [mm] \not=c\,,$ [/mm] beide aus dem [mm] $\IR^n$ [/mm] so, dass sowohl
[mm] $d(a,b)=\text{dist}(a,K)\,$ [/mm] als auch [mm] $d(a,c)=\text{dist}(a,K)$ [/mm] gelte. Dann weißt Du schonmal,
dass $[b,c] [mm] \subseteq [/mm] K$...

Überlegen kann man sich schonmal: $d(a,b)=d(a,c) [mm] \le d(a,\;(1-t)b+t*c)$ [/mm] für alle $0 [mm] \le [/mm] t [mm] \le [/mm] 1$.

Und hinaus wollen wir etwa auf sowas: [mm] $d(b,c)=0\,.$ [/mm]

Vielleicht kann man dann ja so anfangen:
Für zunächst noch nicht näher bestimmtest $0 [mm] \le [/mm] t [mm] \le [/mm] 1$ gilt
$$d(b,c)=d(b, [mm] \;(1-t)b+t*c)+d((1-t)*b+t*c,\;c)\,.$$ [/mm]
(Eigentlich stünde hier [mm] $\le\,,$ [/mm] aber "beachte die Geometrie": Wir haben die
Strecke $[b,c] [mm] \subseteq \IR^n$ [/mm] und einen Punkt, der auf dieser Strecke liegt...)

Vielleicht bekommt man ja schon einen Widerspruch, wenn man hier $t=1/2$
wählt und sich dann [mm] $d(a,\;(1-t)b+t*c)=\|a-\tfrac{1}{2}b-\tfrac{1}{2}c\|$ [/mm] anguckt...

Versprechen kann ich nichts. Falls meine Übrlegungen oben so gar nichts
bringen sollten: Manchmal hilft aber auch eine Skizze. Mit der kannst Du
dann selbst nochmal ein paar kleine Überlegungen anstellen.
Beachtenswert ist sicher auch, dass hier sowas wie die
Parallelogrammgleichung und der Satz des Pythagoras gelten. Ob man das
hier nun gut verwenden kann, das weiß ich nicht...

Ich stelle die Frage mal nur auf "halb beantwortet"!

Gruß,
  Marcel

Bezug
                
Bezug
Satz vom Maximum und Mini: Frage (beantwortet)
Status: (Frage) beantwortet Status 
Datum: 21:51 Mo 03.06.2013
Autor: Joker08


> na, ich kenne die Aufgabe jetzt auch nicht auswendig und
> ich habe sie mir
>  auch nicht neu und zu Ende überlegt, aber was liegt denn
> näher, als hier
>  so anzufangen:
>  Angenommen, es gäbe zwei Punkte [mm]b \not=c\,,[/mm] beide aus dem
> [mm]\IR^n[/mm] so, dass sowohl
>  [mm]d(a,b)=\text{dist}(a,K)\,[/mm] als auch [mm]d(a,c)=\text{dist}(a,K)[/mm]
> gelte. Dann weißt Du schonmal,
>  dass [mm][b,c] \subseteq K[/mm]...

Okay das leuchtet ein.
  

> Überlegen kann man sich schonmal: [mm]d(a,b)=d(a,c) \le d(a,\;(1-t)b+t*c)[/mm]
> für alle [mm]0 \le t \le 1[/mm].

Das ist die dreiecksungleichung, oder sehe ich das falsch ?

>  
> Und hinaus wollen wir etwa auf sowas: [mm]d(b,c)=0\,.[/mm]
>  
> Vielleicht kann man dann ja so anfangen:
>  Für zunächst noch nicht näher bestimmtest [mm]0 \le t \le 1[/mm]
> gilt
>  [mm]d(b,c)=d(b, \;(1-t)b+t*c)+d((1-t)*b+t*c,\;c)\,.[/mm]

> (Eigentlich stünde hier [mm]\le\,,[/mm] aber "beachte die
> Geometrie": Wir haben die
>  Strecke [mm][b,c] \subseteq \IR^n[/mm] und einen Punkt, der auf
> dieser Strecke liegt...)
>  
> Vielleicht bekommt man ja schon einen Widerspruch, wenn man
> hier [mm]t=1/2[/mm]
>  wählt und sich dann
> [mm]d(a,\;(1-t)b+t*c)=\|a-\tfrac{1}{2}b-\tfrac{1}{2}c\|[/mm]
> anguckt...

Also wenn ich mir das ganze mit t=1/2 anschaue bringt mich das so nicht weiter. Ich hab mir das ganze mal allgemeiner angeschaut, also:


[mm] d(a,\;(1-t)b+t\cdot{}c)=\|a-[(1-t)b+t\cdot [/mm] c] [mm] \|=\|a-(b-bt+ct) \| [/mm]

[mm] =\|a-b+bt-ct\| \leq \|a-b\|+\|bt-ct\| [/mm] = [mm] \|a-b\|+t \|b-c\|. [/mm]

Für beliebig kleine t, würd ich den rechten teil, auch belieigt klein bekommen. Nur der linke teil würde noch stören.

denn wenn ich das richtig verstanden habe soll ich ja zeigen dass d(b,c)=0 gilt,  da dann b=c gelten muss im widerspruch zur annahme die punkte seien ungleich. Nur funktioniert nicht so richtig.

> Versprechen kann ich nichts. Falls meine Übrlegungen oben
> so gar nichts
>  bringen sollten: Manchmal hilft aber auch eine Skizze. Mit
> der kannst Du
>  dann selbst nochmal ein paar kleine Überlegungen
> anstellen.
> Beachtenswert ist sicher auch, dass hier sowas wie die
> Parallelogrammgleichung und der Satz des Pythagoras gelten.
> Ob man das
>  hier nun gut verwenden kann, das weiß ich nicht...
>  
> Ich stelle die Frage mal nur auf "halb beantwortet"!

Wieso pythagoras ? Also auch skizzen kann ich mir nicht wirklich anfertigen, da ich ganricht so richtig weiss was ich da zeichnen sollte.

> Gruß,
>    Marcel


Bezug
                        
Bezug
Satz vom Maximum und Mini: Antwort
Status: (Antwort) fertig Status 
Datum: 23:01 Mo 03.06.2013
Autor: Marcel

Hallo,

auch gerade nur kurz. Und nach wie vor: Zu Deiner Aufgabe habe ich mir
noch nichts weiter überlegt. Von daher kann ich Dir weder eine Musterlösung
geben, noch sagen, ob das, was wir hier machen, zielführend ist. Aber ich
hoffe mal, dass es nicht einfach nur gar nichts bringt. ;-)

> > na, ich kenne die Aufgabe jetzt auch nicht auswendig und
> > ich habe sie mir
>  >  auch nicht neu und zu Ende überlegt, aber was liegt
> denn
> > näher, als hier
>  >  so anzufangen:
>  >  Angenommen, es gäbe zwei Punkte [mm]b \not=c\,,[/mm] beide aus
> dem
> > [mm]\IR^n[/mm] so, dass sowohl
>  >  [mm]d(a,b)=\text{dist}(a,K)\,[/mm] als auch
> [mm]d(a,c)=\text{dist}(a,K)[/mm]
> > gelte. Dann weißt Du schonmal,
>  >  dass [mm][b,c] \subseteq K[/mm]...
>  
> Okay das leuchtet ein.
>    
> > Überlegen kann man sich schonmal: [mm]d(a,b)=d(a,c) \le d(a,\;(1-t)b+t*c)[/mm]
> > für alle [mm]0 \le t \le 1[/mm].
>  
> Das ist die dreiecksungleichung, oder sehe ich das falsch
> ?

Das siehst Du falsch. Sowohl [mm] $d(a,b)\,$ [/mm] als auch [mm] $d(a,c)\,$ [/mm] sind ja [mm] $=\text{dist}(a,K)\,.$ [/mm] Dann gilt
natürlich für alle Elemente [mm] $k\,$ [/mm] aus [mm] $K\,,$ [/mm] dass $d(a,b)=d(a,c) [mm] \le [/mm] d(a,k)$ gelten muss.
Und die Elemente $(1-t)*b+t*c$ mit $0 [mm] \le [/mm] t [mm] \le [/mm] 1$ gehören wegen der Konvexität von [mm] $K\,$ [/mm] halt zu [mm] $K\,:$ [/mm]
$$(1-t)*b+t*c [mm] \in [/mm] K [mm] \text{ für jedes }0 \le [/mm] t [mm] \le 1\,.$$ [/mm]
  

> >  

> > Und hinaus wollen wir etwa auf sowas: [mm]d(b,c)=0\,.[/mm]
>  >  
> > Vielleicht kann man dann ja so anfangen:
>  >  Für zunächst noch nicht näher bestimmtest [mm]0 \le t \le 1[/mm]
> > gilt
>  >  [mm]d(b,c)=d(b, \;(1-t)b+t*c)+d((1-t)*b+t*c,\;c)\,.[/mm]
>  
> > (Eigentlich stünde hier [mm]\le\,,[/mm] aber "beachte die
> > Geometrie": Wir haben die
>  >  Strecke [mm][b,c] \subseteq \IR^n[/mm] und einen Punkt, der auf
> > dieser Strecke liegt...)
>  >  
> > Vielleicht bekommt man ja schon einen Widerspruch, wenn man
> > hier [mm]t=1/2[/mm]
>  >  wählt und sich dann
> > [mm]d(a,\;(1-t)b+t*c)=\|a-\tfrac{1}{2}b-\tfrac{1}{2}c\|[/mm]
> > anguckt...
>  
> Also wenn ich mir das ganze mit t=1/2 anschaue bringt mich
> das so nicht weiter. Ich hab mir das ganze mal allgemeiner
> angeschaut, also:
>  
>
> [mm]d(a,\;(1-t)b+t\cdot{}c)=\|a-[(1-t)b+t\cdot[/mm] c]
> [mm]\|=\|a-(b-bt+ct) \|[/mm]
>  
> [mm]=\|a-b+bt-ct\| \leq \|a-b\|+\|bt-ct\|[/mm] = [mm]\|a-b\|+t \|b-c\|.[/mm]
>  
> Für beliebig kleine t, würd ich den rechten teil, auch
> belieigt klein bekommen. Nur der linke teil würde noch
> stören.
>
> denn wenn ich das richtig verstanden habe soll ich ja
> zeigen dass d(b,c)=0 gilt,  da dann b=c gelten muss im
> widerspruch zur annahme die punkte seien ungleich. Nur
> funktioniert nicht so richtig.
>
> > Versprechen kann ich nichts. Falls meine Übrlegungen oben
> > so gar nichts
>  >  bringen sollten: Manchmal hilft aber auch eine Skizze.
> Mit
> > der kannst Du
>  >  dann selbst nochmal ein paar kleine Überlegungen
> > anstellen.
> > Beachtenswert ist sicher auch, dass hier sowas wie die
> > Parallelogrammgleichung und der Satz des Pythagoras gelten.
> > Ob man das
>  >  hier nun gut verwenden kann, das weiß ich nicht...
>  >  
> > Ich stelle die Frage mal nur auf "halb beantwortet"!
>  
> Wieso pythagoras ? Also auch skizzen kann ich mir nicht
> wirklich anfertigen, da ich ganricht so richtig weiss was
> ich da zeichnen sollte.

Na, skizziere was im [mm] $\IR^2,$ [/mm] mit sehr speziellen konvexen Mengen mal
meinetwegen:
Der Kreis ist so ziemlich die speziellste konvexe Menge. Aber auch ein
Quadrat kann man hernehmen. Oder auch eine Ellipse...

Im Prinzip kannst Du eine beschränkte konvexe Menge des [mm] $\IR^2$ [/mm] auch "sehr zufällig
skizzieren", indem Du Dir dann nur die Randlinie anguckst, wenn die Menge
zusammenhängend ist.

Und dann betrachtest Du halt mal einen Punkt [mm] $a\,$ [/mm] außerhalb dieser Menge [mm] $K\,$ [/mm] ...

Jetzt nimmst Du an, es gäbe zwei Punkte in [mm] $K\,,$ [/mm] deren Abstand zu $a [mm] \notin [/mm] K$ gleich [mm] $\text{dist}(a,K)$ [/mm] ist...

Wenn ich nachher oder morgen mal Zeit habe, schaue ich mir das Ganze
vielleicht mal genauer an. Ich glaube eigentlich nicht, dass man hier
großartige Tricks braucht, um [mm] $\|b-c\|=0$ [/mm] zu folgern...

Gruß,
  Marcel

Bezug
        
Bezug
Satz vom Maximum und Mini: Antwort
Status: (Antwort) fertig Status 
Datum: 16:39 Di 04.06.2013
Autor: tobit09

Hallo zusammen,


eine Möglichkeit (die auf Marcels Überlegungen beruht):


Seien $b$ und $c$ nächste Punkte zu $a$ in $K$.


Betrachte die Funktion

     [mm] $f\colon[0,1]\to\IR,\quad t\mapsto d(a,(1-t)b+tc)^2$. [/mm]

Überlege, dass [mm] $f(t)\ge [/mm] f(0)=f(1)$ für alle [mm] $t\in[0,1]$ [/mm] gilt.


Rechne mithilfe der Definition des euklidischen Abstands nach, dass $f$ die Form

     [mm] $f(t)=d(b,c)^2t^2+\lambda t+\mu$ [/mm]

für gewisse [mm] $\lambda,\mu\in\IR$ [/mm] besitzt.


Wäre nun [mm] $d(b,c)\not=0$, [/mm] so wäre der Graph von f die Einschränkung einer nach oben geöffneten Parabel.

Wegen $f(0)=f(1)$ müsste deren Scheitelstelle [mm] $t_0$ [/mm] echt zwischen $0$ und $1$ liegen.

Also wäre [mm] $f(t_0)
Also doch $d(b,c)=0$.


Viele Grüße
Tobias

Bezug
                
Bezug
Satz vom Maximum und Mini: Frage (beantwortet)
Status: (Frage) beantwortet Status 
Datum: 20:51 Di 04.06.2013
Autor: Joker08


> Hallo zusammen,
>  
>
> eine Möglichkeit (die auf Marcels Überlegungen beruht):
>  
>
> Seien [mm]b[/mm] und [mm]c[/mm] nächste Punkte zu [mm]a[/mm] in [mm]K[/mm].
>  
>
> Betrachte die Funktion
>  
> [mm]f\colon[0,1]\to\IR,\quad t\mapsto d(a,(1-t)b+tc)^2[/mm].

Okay, darf ich das denn einfach so ? Denn K ist ja irgend eine kompakte Menge, die konvex ist. Natürlich gilts dann auch für [0,1].

> Überlege, dass [mm]f(t)\ge f(0)=f(1)[/mm] für alle [mm]t\in[0,1][/mm]
> gilt.

Okay [mm] f(0)=(a-b)^2 [/mm]

[mm] f(1)=(a-c)^2 [/mm]

Da ja b,c nähste punkte ist d(a,b)=d(a,c), also auch f(0)=f(1).

Da für d(a,b)=d(a,c)=dist d(a,K) gilt nimmt die funktion f ihr minimum bei f(0)=f(1) an. Nach dem satz vom Maximum und Minimum nimmt f natürlich auch sein maximum auf dem Intervall I:=[0,1] an. Also muss die funktion dort für alle [mm] t\in [/mm] (0,1)  größer als f(0)=f(1) sein. Was schon alleine aus dem minimum folgt, aber naja :D


>
> Rechne mithilfe der Definition des euklidischen Abstands
> nach, dass [mm]f[/mm] die Form
>  
> [mm]f(t)=d(b,c)^2t^2+\lambda t+\mu[/mm]
>  
> für gewisse [mm]\lambda,\mu\in\IR[/mm] besitzt.

Also ich habs mal probiert:

Für [mm] t\in [/mm] (0,1) gilt dann:

[mm] f(t)=d(a,(1-t)b+tc)^2= |a-((1-t)b+tc)|^2 [/mm] = [mm] |a-(b-bt+tc)|^2 [/mm]

= [mm] |a-b+bt-ct|^2 [/mm]

[mm] =(a-b+t(b-c))^2 [/mm] = (a-b+t(b-c))(a-b+t(b-c)) = [mm] (a-b)^2+2t(b-c)+t^2(b-c)^2 [/mm]

Dann hab ich ja schonmal [mm] (a-b)^2+2t(b-c)+t^2d(b,c)^2 [/mm]

aber was nun genau [mm] \lambda, \mu [/mm] werden soll.

Es kann sich ja nur um den teil handeln:

[mm] (a-b)^2+2t(b-c) [/mm]

Das könnte ich noch umschreiben zu:

[mm] a^2-2ab+b^2+2t(b-c) [/mm]

Aber ich sehe jetzt nicht genau was dort [mm] \lamda, \mu [/mm] sein sollte. Ich hab dort ja auch noch (b-c) drin, was eher schlecht ist.




>
> Wäre nun [mm]d(b,c)\not=0[/mm], so wäre der Graph von f die
> Einschränkung einer nach oben geöffneten Parabel.
>  
> Wegen [mm]f(0)=f(1)[/mm] müsste deren Scheitelstelle [mm]t_0[/mm] echt
> zwischen [mm]0[/mm] und [mm]1[/mm] liegen.
>  
> Also wäre [mm]f(t_0)
> alle [mm]t\in[0,1][/mm].
>  
> Also doch [mm]d(b,c)=0[/mm].
>  
>
> Viele Grüße
>  Tobias


Bezug
                        
Bezug
Satz vom Maximum und Mini: Antwort
Status: (Antwort) fertig Status 
Datum: 21:05 Di 04.06.2013
Autor: Marcel

Hallo Joker!

> > Hallo zusammen,
>  >  
> >
> > eine Möglichkeit (die auf Marcels Überlegungen beruht):
>  >  
> >
> > Seien [mm]b[/mm] und [mm]c[/mm] nächste Punkte zu [mm]a[/mm] in [mm]K[/mm].
>  >  
> >
> > Betrachte die Funktion
>  >  
> > [mm]f\colon[0,1]\to\IR,\quad t\mapsto d(a,(1-t)b+tc)^2[/mm].
>  
> Okay, darf ich das denn einfach so ? Denn K ist ja irgend
> eine kompakte Menge, die konvex ist. Natürlich gilts dann
> auch für [0,1].

?? Natürlich darfst Du das:
Wegen $t [mm] \in [/mm] [0,1]$ ist ja $(1-t)b+tc [mm] \in K\,,$ [/mm] weil $b,c [mm] \in [/mm] K$ sind und [mm] $K\,$ [/mm] konvex ist.

Was meinst Du denn mit "Natürlich gilt's dann auch für [0,1]."? Alleine, wenn
ich solch' einen inhaltslosen Satz lese, weiß ich, dass Dir da einiges noch
immer nicht klar ist.

Ich mach's jetzt mal ganz klar, hoffe ich:
Für unsere $b,c [mm] \in [/mm] K$ definieren wir

    [mm] $x_{b,c}\colon [/mm] [0,1] [mm] \to [/mm] K$

durch $[0,1] [mm] \ni [/mm] t [mm] \mapsto x_{b,c}(t):=(1-t)*b+t*c \in [/mm] K [mm] \subseteq \IR^n$ [/mm] ($t [mm] \in [/mm] [0,1]$).

Die Behauptung, dass [mm] $x_{b,c}$ [/mm] nur Werte in [mm] $K\,$ [/mm] annimmt, die wir bei der
Definition explizit mitformuliert haben, begründet sich mit dem, was ich oben
gesagt habe: Wegen $b,c [mm] \in [/mm] K$ folgt $(1-t)*b+t*c [mm] \in [/mm] K$ für alle $t [mm] \in [0,1]\,,$ [/mm] weil [mm] $K\,$ [/mm]
konvex ist.

Nun definieren wir
$$f [mm] \colon [/mm] [0,1] [mm] \to \IR$$ [/mm]
durch [mm] $f(t):=d(a,\;x_{b,c}(t))^\red{\;2}\,.$ [/mm]

Dabei ist $d [mm] \colon \IR^n \times \IR^n \to \IR$ [/mm] die durch die euklidische Norm auf dem [mm] $\IR^n$ [/mm]
induzierte Metrik.

Weiter ist ja $a [mm] \in \IR^n$ [/mm] und für jedes $t [mm] \in [/mm] [0,1]$ ist ja [mm] $x_{b,c}(t) \in [/mm] K [mm] \subseteq \IR^n\,.$ [/mm]

Gruß,
  Marcel

Bezug
                        
Bezug
Satz vom Maximum und Mini: Mitteilung
Status: (Mitteilung) Reaktion unnötig Status 
Datum: 21:09 Di 04.06.2013
Autor: Marcel

Hallo,

auch kurz hierzu:

> [mm]f(t)=d(a,(1-t)b+tc)^2= |a-((1-t)b+tc)|^2[/mm] = [mm]|a-(b-bt+tc)|^2[/mm]
>  
> = [mm]|a-b+bt-ct|^2[/mm]
>  
> [mm]=(a-b+t(b-c))^2[/mm]

Die Elemente $a,b,c$ sind Elemente des [mm] $\IR^n\,.$ [/mm] Was verstehst Du da genau
unter [mm] $p^2$ [/mm] für $p [mm] \in \IR^n$? [/mm]

Gruß,
  Marcel

Bezug
                                
Bezug
Satz vom Maximum und Mini: Frage (beantwortet)
Status: (Frage) beantwortet Status 
Datum: 23:46 Di 04.06.2013
Autor: Joker08


> Hallo,
>  
> auch kurz hierzu:
>  
> > [mm]f(t)=d(a,(1-t)b+tc)^2= |a-((1-t)b+tc)|^2[/mm] = [mm]|a-(b-bt+tc)|^2[/mm]
>  >  
> > = [mm]|a-b+bt-ct|^2[/mm]
>  >  
> > [mm]=(a-b+t(b-c))^2[/mm]
>
> Die Elemente [mm]a,b,c[/mm] sind Elemente des [mm]\IR^n\,.[/mm] Was verstehst
> Du da genau
>  unter [mm]p^2[/mm] für [mm]p \in \IR^n[/mm]?

Ah darauf hatte ich garnicht geachtet.
Für [mm] p^2\in R^n [/mm] hab ich dann

[mm] \vektor{p_1 \\ p_2 \\...\\p_n}^2 [/mm] ?



> Gruß,
>    Marcel


Bezug
                                        
Bezug
Satz vom Maximum und Mini: Antwort
Status: (Antwort) fertig Status 
Datum: 01:20 Mi 05.06.2013
Autor: tobit09

Hallo Joker,


> > Die Elemente [mm]a,b,c[/mm] sind Elemente des [mm]\IR^n\,.[/mm] Was verstehst
> > Du da genau
>  >  unter [mm]p^2[/mm] für [mm]p \in \IR^n[/mm]?
>  
> Ah darauf hatte ich garnicht geachtet.
>  Für [mm]p^2\in R^n[/mm] hab ich dann
>  
> [mm]\vektor{p_1 \\ p_2 \\...\\p_n}^2[/mm] ?

Und was meinst du mit [mm] $\vektor{p_1 \\ p_2 \\...\\p_n}^2$? [/mm]

Wenn du nicht erklären kannst, was du mit dieser Schreibweise meinst, solltest du sie nicht verwenden. Arbeite stattdessen mit der Definition des euklidischen Abstands.


Viele Grüße
Tobias

Bezug
                        
Bezug
Satz vom Maximum und Mini: Antwort
Status: (Antwort) fertig Status 
Datum: 01:57 Mi 05.06.2013
Autor: tobit09

Hallo Joker,


> > Seien [mm]b[/mm] und [mm]c[/mm] nächste Punkte zu [mm]a[/mm] in [mm]K[/mm].
>  >  
> >
> > Betrachte die Funktion
>  >  
> > [mm]f\colon[0,1]\to\IR,\quad t\mapsto d(a,(1-t)b+tc)^2[/mm].
>  
> Okay, darf ich das denn einfach so ? Denn K ist ja irgend
> eine kompakte Menge, die konvex ist. Natürlich gilts dann
> auch für [0,1].

Wie Marcel schon schrieb: Warum sollte es verboten sein, diese Funktion zu betrachten?

(K kommt in der Definition dieser Funktion übrigens gar nicht vor.)


> > Überlege, dass [mm]f(t)\ge f(0)=f(1)[/mm] für alle [mm]t\in[0,1][/mm]
> > gilt.
>  
> Okay [mm]f(0)=(a-b)^2[/mm]
>  
> [mm]f(1)=(a-c)^2[/mm]

Wenn du Doppel-Striche [mm] $\|$ [/mm] für die euklidische Norm anstelle der Klammern verwendest, stimmt es.

> Da ja b,c nähste punkte ist d(a,b)=d(a,c), also auch
> f(0)=f(1).

[ok]

> Da für d(a,b)=d(a,c)=dist d(a,K) gilt nimmt die funktion f
> ihr minimum bei f(0)=f(1) an.

Ja. Denn mit Marcels Notationen gilt [mm] $f(t):=d(a,\;x_{b,c}(t))^2$ [/mm] und für alle [mm] $t\in[0,1]$ [/mm] gilt [mm] $x_{b,c}(t)\in [/mm] K$.

> Nach dem satz vom Maximum und
> Minimum nimmt f natürlich auch sein maximum auf dem
> Intervall I:=[0,1] an.

Das brauchst du hier zwar nicht, aber das stimmt.

> Also muss die funktion dort für
> alle [mm]t\in[/mm] (0,1)  größer als f(0)=f(1) sein.

Du meinst wohl "größer gleich" statt "größer".

Alleine daraus, dass $f$ irgendwo auf $[0,1]$ sein Maximum annimmt, folgt noch lange nicht, dass die Funktion an allen Stellen [mm] $t\in(0,1)$ [/mm] Werte größer gleich $f(0)=f(1)$ annimmt.


> > Rechne mithilfe der Definition des euklidischen Abstands
> > nach, dass [mm]f[/mm] die Form
>  >  
> > [mm]f(t)=d(b,c)^2t^2+\lambda t+\mu[/mm]
>  >  
> > für gewisse [mm]\lambda,\mu\in\IR[/mm] besitzt.
>  
> Also ich habs mal probiert:
>  
> Für [mm]t\in[/mm] (0,1) gilt dann:
>  
> [mm]f(t)=d(a,(1-t)b+tc)^2= |a-((1-t)b+tc)|^2[/mm] = [mm]|a-(b-bt+tc)|^2[/mm]
>  
> = [mm]|a-b+bt-ct|^2[/mm]

Mit den Betragsstrichen meinst du die euklidische Norm? Dann passt es.

> [mm]=(a-b+t(b-c))^2[/mm] = (a-b+t(b-c))(a-b+t(b-c)) =

Setze Normstriche [mm] $\|$ [/mm] statt der äußeren Klammern!

> [mm](a-b)^2+2t(b-c)+t^2(b-c)^2[/mm]

(Wo ist der Term (a-b) in der Mitte geblieben?)

Wenn du

     [mm] "$(a-b)^2+2t(a-b)(b-c)+t^2(b-c)^2$" [/mm]

schreiben möchtest, musst du erklären können, was du mit $(a-b)(b-c)$ meinst und warum diese Gleichheit gilt.

Spätestens hier solltest du mal die Definition der euklidischen Norm ins Spiel bringen:

     [mm] $\|(x_1,\ldots,x_n)\|:=\wurzel{\sum_{i=1}^n (x_i)^2}$ [/mm]

Dann brauchst du die Schreibweisen, die du anscheinend selbst nicht erklären kannst, gar nicht.

  

> Dann hab ich ja schonmal [mm](a-b)^2+2t(b-c)+t^2d(b,c)^2[/mm]
>  
> aber was nun genau [mm]\lambda, \mu[/mm] werden soll.

Das können wir klären, wenn du f(t) korrekt ausgerechnet hast.

Letztlich spielen die genauen Werte von [mm] $\lambda$ [/mm] und [mm] $\mu$ [/mm] aber auch keine Rolle: Für die weitere Argumentation ist nur wichtig, dass $f$ im Falle [mm] $d(b,c)\not=0$ [/mm] eine quadratische Funktion (bzw. deren Einschränkung auf [0,1]) ist, deren Scheitelpunkt ein Minimum ist.


Viele Grüße
Tobias

Bezug
                
Bezug
Satz vom Maximum und Mini: Scheitelstelle
Status: (Mitteilung) Reaktion unnötig Status 
Datum: 00:45 Fr 07.06.2013
Autor: Marcel

Hallo,

> Wegen [mm]f(0)=f(1)[/mm] müsste deren Scheitelstelle [mm]t_0[/mm] echt
> zwischen [mm]0[/mm] und [mm]1[/mm]Eingabefehler: "\left" und "\right" müssen immer paarweise auftreten, es wurde aber ein Teil ohne Entsprechung gefunden (siehe rote Markierung)

liegen.

auch, wenn es mehr oder weniger trivial ist: Man kann sich leicht überlegen,
dass diese Scheitelstelle genau an $t=1/2\,$ liegen muss.

Allgemein gilt: Ist $f(x)=ax^2+bx+c$ ($a \not=0$) und sind $r \not=s$ mit $f(r)=f(s)\,,$ so folgt, dass die
Scheitelstelle an
$$\frac{r+s}{2}$$
liegt.

Beweis:
Wir berechnen zunächst die Scheitelstelle auf zwei Arten (eine davon
würde uns reichen; die erste machen wir "nur zu Übungs- und
Wiederholungszwecken"):

(1) $$ax^2+bx+c=a((x+\tfrac{b}{2a})^2-\tfrac{b^2}{4a^2})+c=a((x+\tfrac{b}{2a})^2)+(c-\tfrac{b^2}{4a})$$
zeigt, dass der Scheitelpunkt gerade
$$(\left.-\,\tfrac{b}{2a}\;\right|\;c-\tfrac{b^2}{4a})$$
ist - die Scheitelstelle ist also $-\;\frac{b}{2a}\,.$

(2) "Einfacher": $x \mapsto f(x)=ax^2+bx+c$ hat genau eine lokale (hier auch globale)
Extremstelle: $f\,'(x)=2ax+b=0 \iff x=\;-\;\tfrac{b}{2a}$ zeigt: die Scheitelstelle ist $-\;\frac{b}{2a}\,.$

Wir haben oben also zu beweisen, dass $(r+s)/2=\;-\;b/(2a)$ gilt.
Beweis dazu:
Aus $f(r)=f(s)$ folgt
$$ar^2+br+c=as^2+bs+c$$
$$\iff a(r+s)(r-s)+b(r-s)=0\,.$$

Wegen $r-s \not=0$ ist letzte Gleichung gleichwertig mit
$$a(r+s)+b=0\,,$$
also
$$\frac{r+s}{2}=\;-\;\frac{b}{2a}\,.$$

Dies beendet den Beweis. $\hfill \Box$

Gruß,
  Marcel

Bezug
        
Bezug
Satz vom Maximum und Mini: Mitteilung
Status: (Mitteilung) Reaktion unnötig Status 
Datum: 03:24 Mi 05.06.2013
Autor: Marcel

Hallo,

was ich hier übrigens auch nochmal ergänzen wollte:

> Sei (X,d) metrischer Raum.
>  
> (a) Zeigen Sie: Für Festes [mm]a\in[/mm] X ist die Abbildung f:
> [mm]X\to \IR[/mm] mit f(x):=d(x,a) stetig.
>  
> (b) Sie K [mm]\subset[/mm] X kompakt. Zeigen Sie, dass es zu jedem
> [mm]a\in[/mm] X einen Punkt [mm]b\in[/mm] K mit d(a,b)=dist(a,K) gibt, also
> einen "nächsten Punkt" zu a in K.

Man nennt dann $b [mm] \in [/mm] K$ "Bestapproximation"! Und ich will auch nicht unerwähnt
lassen:
Ist [mm] $(V,\|.\|)\,$ [/mm] ein normierter Raum und $K [mm] \subseteq [/mm] V$ konvex, so ist die Menge der
Bestapproximationen konvex.

Beweis:
Seien $b,c [mm] \in K\,,$ [/mm] $a [mm] \in [/mm] V$ mit [mm] $\|a-b\|=\|a-c\|\le \|a-k\|$ [/mm] für $k [mm] \in K\,.$ [/mm] Dann gilt
für alle $t [mm] \in [/mm] [0,1]$
[mm] $$\|a-(t*b+(1-t)*c)\|=\|t*(a-b)+(1-t)*(a-c)\| \le t*\|a-b\|+(1-t)*\|a-c\|=\|a-b\|\,.$$ [/mm]
Ich denke eigentlich, dass man solch' eine Erkenntnis auch in Aufgabenteil
3 hätte benutzen können. Nicht, dass ich Tobis Beweis in seiner Eleganz
schmälern will, aber dennoch werde ich vielleicht mal noch nach einem
alternativen Beweis suchen ^^

Gruß,
  Marcel

Bezug
        
Bezug
Satz vom Maximum und Mini: Antwort
Status: (Antwort) fertig Status 
Datum: 11:56 Mi 05.06.2013
Autor: Marcel

Hallo,

hier nur mal eine alternative Beweisskizze zu (c) (eigentlich eher ein
unvollständiger Beweis; man muss halt "geometrische Aspekte" noch
formal beweisen):

> Sei (X,d) metrischer Raum.
>  
> (a) Zeigen Sie: Für Festes [mm]a\in[/mm] X ist die Abbildung f:
> [mm]X\to \IR[/mm] mit f(x):=d(x,a) stetig.
>  
> (b) Sie K [mm]\subset[/mm] X kompakt. Zeigen Sie, dass es zu jedem
> [mm]a\in[/mm] X einen Punkt [mm]b\in[/mm] K mit d(a,b)=dist(a,K) gibt, also
> einen "nächsten Punkt" zu a in K.
>  
> (c) Dieser ist sogar eindeutig, wenn [mm]X=\IR^n[/mm] und
> zusätzlich K konvex ist.

Nehmen wir an, es gäbe zwei solcher Punkte namens $b,c [mm] \in [/mm] K$ (also alles wie
gehabt), und es wäre [mm] $\|b-c\| \not=0\,.$ [/mm] Wie hier (klick!) schon bemerkt ist dann
auch automatisch die Strecke [mm] $\overline{b,\,c}:=\{t*b+(1-t)*c:\;\; 0 \le t \le 1\} \subseteq [/mm] K$ eine Menge
bestapproximierender Elemente an [mm] $a\,.$ [/mm]

Für alle $x [mm] \in \overline{b,\,c}$ [/mm] folgt dann
[mm] $$d(a,\,x)=d(a,K)\,.$$ [/mm]

Ist [mm] $d(a,K)=0\,,$ [/mm] so erhalten wir natürlich sofort einen Widerspruch (welchen?).

Ohne Einschränkung sei also $d(a,K) [mm] \not=0\,.$ [/mm] Ich könnte jetzt mit dem
"Rand einer Kugel" argumentieren, aber ich mache es anders:
Die Vektoren [mm] $a-c\,$ [/mm] und [mm] $a-b\,$ [/mm] müssen linear unabhängig sein (warum?).
Damit ist
[mm] $$E:=\{a+r*(a-c)+s*(a-b):\;\;r,s \in \IR\}$$ [/mm]
eine Ebene des [mm] $\IR^n\,.$ [/mm] Betrachten wir nun mit $R:=d(a,K) > 0$ den Rand der Kugel
mit Radius [mm] $R\,$ [/mm] um [mm] $a\,,$ [/mm] also
[mm] $$\partial K_R(a)=\{y \in \IR^n:\;\;\|y-a\|=R\}$$ [/mm]
und betrachten nun
[mm] $$\text{Kreis}:=\text{Kreis}_R(a)=\partial K_R(a) \cap E\,,$$ [/mm]
so ist [mm] $\text{Kreis} \subseteq [/mm] E$ in der Tat ein Kreis in [mm] $E\,,$ [/mm] nämlich der Kreis mit
Mittelpunkt [mm] $a\,$ [/mm] und Radius [mm] $R\,.$ [/mm]

Wegen $d(a,x)=R$ für alle $x [mm] \in \overline{b,\,c} \subseteq [/mm] E$ muss dann die Strecke [mm] $\overline{b,\,c}$ [/mm] gleichzeitig auch ein
Teil der Randlinie dieses Kreises sein.
Das kann aber nicht sein (und das ist noch ein wichtiger Punkt: Das sollte
man noch formal beweisen; dazu reicht es aber, sich o.E. diese Aussage im
[mm] $\IR^2$ [/mm] einfach klarzumachen, wobei man auch o.E. den Kreis mit Radius
1 und dem Ursprung als Mittelpunkt wählen kann)!

Gruß,
  Marcel

Bezug
Ansicht: [ geschachtelt ] | ^ Forum "Reelle Analysis mehrerer Veränderlichen"  | ^^ Alle Foren  | ^ Forenbaum  | Materialien


^ Seitenanfang ^
www.matheforum.net
[ Startseite | Forum | Wissen | Kurse | Mitglieder | Team | Impressum ]